In need of interpretation of a ordered-subset related problem.












0












$begingroup$


''Given a set $S_n = {1, 2, 3, ldots, n}$, we define a preference list to be an ordered subset of $S_n$. Let $P_n$ be the number of preference lists of $S_n$. Show that for positive integers $n > m$, $P_n - P_m$ is divisible by $n - m$.



Note: the empty set and $S_n$ are subsets of $S_n$.''



Can anyone help me out to make me understand the problem ? (giving an explained example of this question will be very good )
Thanks in advance :)










share|cite|improve this question









$endgroup$

















    0












    $begingroup$


    ''Given a set $S_n = {1, 2, 3, ldots, n}$, we define a preference list to be an ordered subset of $S_n$. Let $P_n$ be the number of preference lists of $S_n$. Show that for positive integers $n > m$, $P_n - P_m$ is divisible by $n - m$.



    Note: the empty set and $S_n$ are subsets of $S_n$.''



    Can anyone help me out to make me understand the problem ? (giving an explained example of this question will be very good )
    Thanks in advance :)










    share|cite|improve this question









    $endgroup$















      0












      0








      0





      $begingroup$


      ''Given a set $S_n = {1, 2, 3, ldots, n}$, we define a preference list to be an ordered subset of $S_n$. Let $P_n$ be the number of preference lists of $S_n$. Show that for positive integers $n > m$, $P_n - P_m$ is divisible by $n - m$.



      Note: the empty set and $S_n$ are subsets of $S_n$.''



      Can anyone help me out to make me understand the problem ? (giving an explained example of this question will be very good )
      Thanks in advance :)










      share|cite|improve this question









      $endgroup$




      ''Given a set $S_n = {1, 2, 3, ldots, n}$, we define a preference list to be an ordered subset of $S_n$. Let $P_n$ be the number of preference lists of $S_n$. Show that for positive integers $n > m$, $P_n - P_m$ is divisible by $n - m$.



      Note: the empty set and $S_n$ are subsets of $S_n$.''



      Can anyone help me out to make me understand the problem ? (giving an explained example of this question will be very good )
      Thanks in advance :)







      combinatorics number-theory






      share|cite|improve this question













      share|cite|improve this question











      share|cite|improve this question




      share|cite|improve this question










      asked Dec 29 '18 at 13:10









      nahin munkarnahin munkar

      1




      1






















          1 Answer
          1






          active

          oldest

          votes


















          0












          $begingroup$

          For example, for $n=3$ the preference lists of ${1,2,3}$ are
          $$ emptyset\
          (1)\
          (2)\
          (3)\
          (1, 2)\
          (2, 1)\
          (1, 3)\
          (3, 1)\
          (2, 3)\
          (3, 2)\
          (1, 2, 3)\
          (1, 3, 2)\
          (2, 1, 3)\
          (2, 3, 1)\
          (3, 1, 2)\
          (3, 2, 1)$$






          share|cite|improve this answer









          $endgroup$














            Your Answer








            StackExchange.ready(function() {
            var channelOptions = {
            tags: "".split(" "),
            id: "69"
            };
            initTagRenderer("".split(" "), "".split(" "), channelOptions);

            StackExchange.using("externalEditor", function() {
            // Have to fire editor after snippets, if snippets enabled
            if (StackExchange.settings.snippets.snippetsEnabled) {
            StackExchange.using("snippets", function() {
            createEditor();
            });
            }
            else {
            createEditor();
            }
            });

            function createEditor() {
            StackExchange.prepareEditor({
            heartbeatType: 'answer',
            autoActivateHeartbeat: false,
            convertImagesToLinks: true,
            noModals: true,
            showLowRepImageUploadWarning: true,
            reputationToPostImages: 10,
            bindNavPrevention: true,
            postfix: "",
            imageUploader: {
            brandingHtml: "Powered by u003ca class="icon-imgur-white" href="https://imgur.com/"u003eu003c/au003e",
            contentPolicyHtml: "User contributions licensed under u003ca href="https://creativecommons.org/licenses/by-sa/3.0/"u003ecc by-sa 3.0 with attribution requiredu003c/au003e u003ca href="https://stackoverflow.com/legal/content-policy"u003e(content policy)u003c/au003e",
            allowUrls: true
            },
            noCode: true, onDemand: true,
            discardSelector: ".discard-answer"
            ,immediatelyShowMarkdownHelp:true
            });


            }
            });














            draft saved

            draft discarded


















            StackExchange.ready(
            function () {
            StackExchange.openid.initPostLogin('.new-post-login', 'https%3a%2f%2fmath.stackexchange.com%2fquestions%2f3055833%2fin-need-of-interpretation-of-a-ordered-subset-related-problem%23new-answer', 'question_page');
            }
            );

            Post as a guest















            Required, but never shown

























            1 Answer
            1






            active

            oldest

            votes








            1 Answer
            1






            active

            oldest

            votes









            active

            oldest

            votes






            active

            oldest

            votes









            0












            $begingroup$

            For example, for $n=3$ the preference lists of ${1,2,3}$ are
            $$ emptyset\
            (1)\
            (2)\
            (3)\
            (1, 2)\
            (2, 1)\
            (1, 3)\
            (3, 1)\
            (2, 3)\
            (3, 2)\
            (1, 2, 3)\
            (1, 3, 2)\
            (2, 1, 3)\
            (2, 3, 1)\
            (3, 1, 2)\
            (3, 2, 1)$$






            share|cite|improve this answer









            $endgroup$


















              0












              $begingroup$

              For example, for $n=3$ the preference lists of ${1,2,3}$ are
              $$ emptyset\
              (1)\
              (2)\
              (3)\
              (1, 2)\
              (2, 1)\
              (1, 3)\
              (3, 1)\
              (2, 3)\
              (3, 2)\
              (1, 2, 3)\
              (1, 3, 2)\
              (2, 1, 3)\
              (2, 3, 1)\
              (3, 1, 2)\
              (3, 2, 1)$$






              share|cite|improve this answer









              $endgroup$
















                0












                0








                0





                $begingroup$

                For example, for $n=3$ the preference lists of ${1,2,3}$ are
                $$ emptyset\
                (1)\
                (2)\
                (3)\
                (1, 2)\
                (2, 1)\
                (1, 3)\
                (3, 1)\
                (2, 3)\
                (3, 2)\
                (1, 2, 3)\
                (1, 3, 2)\
                (2, 1, 3)\
                (2, 3, 1)\
                (3, 1, 2)\
                (3, 2, 1)$$






                share|cite|improve this answer









                $endgroup$



                For example, for $n=3$ the preference lists of ${1,2,3}$ are
                $$ emptyset\
                (1)\
                (2)\
                (3)\
                (1, 2)\
                (2, 1)\
                (1, 3)\
                (3, 1)\
                (2, 3)\
                (3, 2)\
                (1, 2, 3)\
                (1, 3, 2)\
                (2, 1, 3)\
                (2, 3, 1)\
                (3, 1, 2)\
                (3, 2, 1)$$







                share|cite|improve this answer












                share|cite|improve this answer



                share|cite|improve this answer










                answered Dec 29 '18 at 14:13









                awkwardawkward

                6,92511026




                6,92511026






























                    draft saved

                    draft discarded




















































                    Thanks for contributing an answer to Mathematics Stack Exchange!


                    • Please be sure to answer the question. Provide details and share your research!

                    But avoid



                    • Asking for help, clarification, or responding to other answers.

                    • Making statements based on opinion; back them up with references or personal experience.


                    Use MathJax to format equations. MathJax reference.


                    To learn more, see our tips on writing great answers.




                    draft saved


                    draft discarded














                    StackExchange.ready(
                    function () {
                    StackExchange.openid.initPostLogin('.new-post-login', 'https%3a%2f%2fmath.stackexchange.com%2fquestions%2f3055833%2fin-need-of-interpretation-of-a-ordered-subset-related-problem%23new-answer', 'question_page');
                    }
                    );

                    Post as a guest















                    Required, but never shown





















































                    Required, but never shown














                    Required, but never shown












                    Required, but never shown







                    Required, but never shown

































                    Required, but never shown














                    Required, but never shown












                    Required, but never shown







                    Required, but never shown







                    Popular posts from this blog

                    Biblatex bibliography style without URLs when DOI exists (in Overleaf with Zotero bibliography)

                    ComboBox Display Member on multiple fields

                    Is it possible to collect Nectar points via Trainline?